Determinant of an n x n matrix












3














I do not know what this kind of matrix is called, it does not really look Circulant, but I tried to do many row and columns operation in order to make it into an upper triangular matrix so the determinant would be the product of the diagonal elements but I couldn't find a way. Any thoughts?



This is the matrix :



$$begin{bmatrix}n&n-1&n-2&cdots&2&1\1&n&n-1&cdots&3&2\1&1&n&cdots&4&3\vdots&vdots&vdots&ddots&vdots&vdots\1&1&1&cdots&n&n-1\1&1&1&cdots&1&lambdaend{bmatrix}$$










share|cite|improve this question









New contributor




Paul Vinur is a new contributor to this site. Take care in asking for clarification, commenting, and answering.
Check out our Code of Conduct.
















  • 1




    It is not clear what you need/want since you even didn't show us the matrix! Try to clarify your question.
    – Sigur
    3 hours ago












  • Do you have a particular matrix in mind, or just an arbitrary $n times n$ matrix?
    – Clive Newstead
    3 hours ago










  • I am sorry, the link to the picture was not included. I added it now.
    – Paul Vinur
    3 hours ago










  • Have you tried expanding over the first column or row for example or calculate the determinant for small values of $n$?
    – Test123
    3 hours ago










  • I did, the determinants for values of n = 3,4,5.. I can sense the pattern I just cannot find the general rule. It would be easier to try simplifying the matrix first. But what do you mean by expanding over the first column?
    – Paul Vinur
    1 hour ago
















3














I do not know what this kind of matrix is called, it does not really look Circulant, but I tried to do many row and columns operation in order to make it into an upper triangular matrix so the determinant would be the product of the diagonal elements but I couldn't find a way. Any thoughts?



This is the matrix :



$$begin{bmatrix}n&n-1&n-2&cdots&2&1\1&n&n-1&cdots&3&2\1&1&n&cdots&4&3\vdots&vdots&vdots&ddots&vdots&vdots\1&1&1&cdots&n&n-1\1&1&1&cdots&1&lambdaend{bmatrix}$$










share|cite|improve this question









New contributor




Paul Vinur is a new contributor to this site. Take care in asking for clarification, commenting, and answering.
Check out our Code of Conduct.
















  • 1




    It is not clear what you need/want since you even didn't show us the matrix! Try to clarify your question.
    – Sigur
    3 hours ago












  • Do you have a particular matrix in mind, or just an arbitrary $n times n$ matrix?
    – Clive Newstead
    3 hours ago










  • I am sorry, the link to the picture was not included. I added it now.
    – Paul Vinur
    3 hours ago










  • Have you tried expanding over the first column or row for example or calculate the determinant for small values of $n$?
    – Test123
    3 hours ago










  • I did, the determinants for values of n = 3,4,5.. I can sense the pattern I just cannot find the general rule. It would be easier to try simplifying the matrix first. But what do you mean by expanding over the first column?
    – Paul Vinur
    1 hour ago














3












3








3







I do not know what this kind of matrix is called, it does not really look Circulant, but I tried to do many row and columns operation in order to make it into an upper triangular matrix so the determinant would be the product of the diagonal elements but I couldn't find a way. Any thoughts?



This is the matrix :



$$begin{bmatrix}n&n-1&n-2&cdots&2&1\1&n&n-1&cdots&3&2\1&1&n&cdots&4&3\vdots&vdots&vdots&ddots&vdots&vdots\1&1&1&cdots&n&n-1\1&1&1&cdots&1&lambdaend{bmatrix}$$










share|cite|improve this question









New contributor




Paul Vinur is a new contributor to this site. Take care in asking for clarification, commenting, and answering.
Check out our Code of Conduct.











I do not know what this kind of matrix is called, it does not really look Circulant, but I tried to do many row and columns operation in order to make it into an upper triangular matrix so the determinant would be the product of the diagonal elements but I couldn't find a way. Any thoughts?



This is the matrix :



$$begin{bmatrix}n&n-1&n-2&cdots&2&1\1&n&n-1&cdots&3&2\1&1&n&cdots&4&3\vdots&vdots&vdots&ddots&vdots&vdots\1&1&1&cdots&n&n-1\1&1&1&cdots&1&lambdaend{bmatrix}$$







linear-algebra matrices determinant






share|cite|improve this question









New contributor




Paul Vinur is a new contributor to this site. Take care in asking for clarification, commenting, and answering.
Check out our Code of Conduct.











share|cite|improve this question









New contributor




Paul Vinur is a new contributor to this site. Take care in asking for clarification, commenting, and answering.
Check out our Code of Conduct.









share|cite|improve this question




share|cite|improve this question








edited 3 hours ago









StubbornAtom

5,26711138




5,26711138






New contributor




Paul Vinur is a new contributor to this site. Take care in asking for clarification, commenting, and answering.
Check out our Code of Conduct.









asked 3 hours ago









Paul Vinur

264




264




New contributor




Paul Vinur is a new contributor to this site. Take care in asking for clarification, commenting, and answering.
Check out our Code of Conduct.





New contributor





Paul Vinur is a new contributor to this site. Take care in asking for clarification, commenting, and answering.
Check out our Code of Conduct.






Paul Vinur is a new contributor to this site. Take care in asking for clarification, commenting, and answering.
Check out our Code of Conduct.








  • 1




    It is not clear what you need/want since you even didn't show us the matrix! Try to clarify your question.
    – Sigur
    3 hours ago












  • Do you have a particular matrix in mind, or just an arbitrary $n times n$ matrix?
    – Clive Newstead
    3 hours ago










  • I am sorry, the link to the picture was not included. I added it now.
    – Paul Vinur
    3 hours ago










  • Have you tried expanding over the first column or row for example or calculate the determinant for small values of $n$?
    – Test123
    3 hours ago










  • I did, the determinants for values of n = 3,4,5.. I can sense the pattern I just cannot find the general rule. It would be easier to try simplifying the matrix first. But what do you mean by expanding over the first column?
    – Paul Vinur
    1 hour ago














  • 1




    It is not clear what you need/want since you even didn't show us the matrix! Try to clarify your question.
    – Sigur
    3 hours ago












  • Do you have a particular matrix in mind, or just an arbitrary $n times n$ matrix?
    – Clive Newstead
    3 hours ago










  • I am sorry, the link to the picture was not included. I added it now.
    – Paul Vinur
    3 hours ago










  • Have you tried expanding over the first column or row for example or calculate the determinant for small values of $n$?
    – Test123
    3 hours ago










  • I did, the determinants for values of n = 3,4,5.. I can sense the pattern I just cannot find the general rule. It would be easier to try simplifying the matrix first. But what do you mean by expanding over the first column?
    – Paul Vinur
    1 hour ago








1




1




It is not clear what you need/want since you even didn't show us the matrix! Try to clarify your question.
– Sigur
3 hours ago






It is not clear what you need/want since you even didn't show us the matrix! Try to clarify your question.
– Sigur
3 hours ago














Do you have a particular matrix in mind, or just an arbitrary $n times n$ matrix?
– Clive Newstead
3 hours ago




Do you have a particular matrix in mind, or just an arbitrary $n times n$ matrix?
– Clive Newstead
3 hours ago












I am sorry, the link to the picture was not included. I added it now.
– Paul Vinur
3 hours ago




I am sorry, the link to the picture was not included. I added it now.
– Paul Vinur
3 hours ago












Have you tried expanding over the first column or row for example or calculate the determinant for small values of $n$?
– Test123
3 hours ago




Have you tried expanding over the first column or row for example or calculate the determinant for small values of $n$?
– Test123
3 hours ago












I did, the determinants for values of n = 3,4,5.. I can sense the pattern I just cannot find the general rule. It would be easier to try simplifying the matrix first. But what do you mean by expanding over the first column?
– Paul Vinur
1 hour ago




I did, the determinants for values of n = 3,4,5.. I can sense the pattern I just cannot find the general rule. It would be easier to try simplifying the matrix first. But what do you mean by expanding over the first column?
– Paul Vinur
1 hour ago










2 Answers
2






active

oldest

votes


















4














This is a rank one update of a triangular matrix. Let $A$ be the matrix in the post and let $B$ be the matrix with entries $b_{ij} = a_{ij} - 1$. Let $e$ be the column vector of 1's. Then $A = ee^T + B$.



Then $det(A) = det(B) + e^TB^{-1}e$. Since $B$ is triangular, $B^{-1}e$ is not hard to find.



This is defined if $lambda ne 1$. For $lambda = 1$, take the limit.






share|cite|improve this answer

















  • 1




    Isn't $det A=det B (1+e^TB^{-1}e)$ ?
    – StubbornAtom
    1 hour ago



















2














Let $M_n$ be your matrix.



Let $eta_n$ be the $ntimes n$ matrix with entry $1$ at the superdiagonal and $0$ 4 elsewhere. If you




  1. Subtract row $k+1$ from row $k$ for $k = 1,2,ldots,n-1$.

    This is equivalent to multiply $M_n$ by $I_n - eta_n$ from the left


  2. Subtract column $k-1$ from column $k$ for $k = n,n-1,ldots,2$ (notice the order of $k$).

    This is equivalent to multiply $(I_n-eta_n)M_n$ by $I_n - eta_n$ from the right.



After you do this, your matrix simplfies to
$$(I_n - eta_n) M_n (I_n - eta_n) =
begin{bmatrix}
n-1&-n&0&cdots&0&0&0\
0&n-1&-n&cdots&0&0&0\
0&0&n-1&ddots&0&0&0\
vdots&vdots&vdots&ddots&ddots&vdots&vdots\
0&0&0&cdots&n-1&-n&0\
0&0&0&cdots&0&n-1&-lambda\
1&0&0&cdots&0&0&lambda-1
end{bmatrix}$$



From this, you can deduce



$$det[M_n] = det[(I_n - eta_n)M_n(I_n - eta_n)]
= (n-1)^{n-1}(lambda-1) + n^{n-2}lambda$$






share|cite|improve this answer























  • This is obviously the correct way since I checked the final answer. But the element "1" in the bottom left is still there and therefore not all elements (except diagonal and superdiagonal) are zeroes, and correct me if I am wrong, wouldn't that mean this is not a triangular matrix and we won't be able to find the det. by just multiplying the diagonal elements? And also I don't quite understand how we got (n^n-2 * λ)
    – Paul Vinur
    6 mins ago












  • @PaulVinur This is not a triangular matrix. Since the entries $a_{ij}$ vanishes unless $j = i text{ or } i+1 pmod n$. When you expand the determinant out completely, among all the $n!$ possible terms in the determinant, only two terms survive. i.e those of the form $prod_{i} a_ii$ and $prod_{} a_{i,i+1}$ ($i+1$ upto modulo $n$) survive. That's why the final expression is a sum of two terms.
    – achille hui
    1 min ago













Your Answer





StackExchange.ifUsing("editor", function () {
return StackExchange.using("mathjaxEditing", function () {
StackExchange.MarkdownEditor.creationCallbacks.add(function (editor, postfix) {
StackExchange.mathjaxEditing.prepareWmdForMathJax(editor, postfix, [["$", "$"], ["\\(","\\)"]]);
});
});
}, "mathjax-editing");

StackExchange.ready(function() {
var channelOptions = {
tags: "".split(" "),
id: "69"
};
initTagRenderer("".split(" "), "".split(" "), channelOptions);

StackExchange.using("externalEditor", function() {
// Have to fire editor after snippets, if snippets enabled
if (StackExchange.settings.snippets.snippetsEnabled) {
StackExchange.using("snippets", function() {
createEditor();
});
}
else {
createEditor();
}
});

function createEditor() {
StackExchange.prepareEditor({
heartbeatType: 'answer',
autoActivateHeartbeat: false,
convertImagesToLinks: true,
noModals: true,
showLowRepImageUploadWarning: true,
reputationToPostImages: 10,
bindNavPrevention: true,
postfix: "",
imageUploader: {
brandingHtml: "Powered by u003ca class="icon-imgur-white" href="https://imgur.com/"u003eu003c/au003e",
contentPolicyHtml: "User contributions licensed under u003ca href="https://creativecommons.org/licenses/by-sa/3.0/"u003ecc by-sa 3.0 with attribution requiredu003c/au003e u003ca href="https://stackoverflow.com/legal/content-policy"u003e(content policy)u003c/au003e",
allowUrls: true
},
noCode: true, onDemand: true,
discardSelector: ".discard-answer"
,immediatelyShowMarkdownHelp:true
});


}
});






Paul Vinur is a new contributor. Be nice, and check out our Code of Conduct.










draft saved

draft discarded


















StackExchange.ready(
function () {
StackExchange.openid.initPostLogin('.new-post-login', 'https%3a%2f%2fmath.stackexchange.com%2fquestions%2f3053935%2fdeterminant-of-an-n-x-n-matrix%23new-answer', 'question_page');
}
);

Post as a guest















Required, but never shown

























2 Answers
2






active

oldest

votes








2 Answers
2






active

oldest

votes









active

oldest

votes






active

oldest

votes









4














This is a rank one update of a triangular matrix. Let $A$ be the matrix in the post and let $B$ be the matrix with entries $b_{ij} = a_{ij} - 1$. Let $e$ be the column vector of 1's. Then $A = ee^T + B$.



Then $det(A) = det(B) + e^TB^{-1}e$. Since $B$ is triangular, $B^{-1}e$ is not hard to find.



This is defined if $lambda ne 1$. For $lambda = 1$, take the limit.






share|cite|improve this answer

















  • 1




    Isn't $det A=det B (1+e^TB^{-1}e)$ ?
    – StubbornAtom
    1 hour ago
















4














This is a rank one update of a triangular matrix. Let $A$ be the matrix in the post and let $B$ be the matrix with entries $b_{ij} = a_{ij} - 1$. Let $e$ be the column vector of 1's. Then $A = ee^T + B$.



Then $det(A) = det(B) + e^TB^{-1}e$. Since $B$ is triangular, $B^{-1}e$ is not hard to find.



This is defined if $lambda ne 1$. For $lambda = 1$, take the limit.






share|cite|improve this answer

















  • 1




    Isn't $det A=det B (1+e^TB^{-1}e)$ ?
    – StubbornAtom
    1 hour ago














4












4








4






This is a rank one update of a triangular matrix. Let $A$ be the matrix in the post and let $B$ be the matrix with entries $b_{ij} = a_{ij} - 1$. Let $e$ be the column vector of 1's. Then $A = ee^T + B$.



Then $det(A) = det(B) + e^TB^{-1}e$. Since $B$ is triangular, $B^{-1}e$ is not hard to find.



This is defined if $lambda ne 1$. For $lambda = 1$, take the limit.






share|cite|improve this answer












This is a rank one update of a triangular matrix. Let $A$ be the matrix in the post and let $B$ be the matrix with entries $b_{ij} = a_{ij} - 1$. Let $e$ be the column vector of 1's. Then $A = ee^T + B$.



Then $det(A) = det(B) + e^TB^{-1}e$. Since $B$ is triangular, $B^{-1}e$ is not hard to find.



This is defined if $lambda ne 1$. For $lambda = 1$, take the limit.







share|cite|improve this answer












share|cite|improve this answer



share|cite|improve this answer










answered 2 hours ago









Hans Engler

10.1k11836




10.1k11836








  • 1




    Isn't $det A=det B (1+e^TB^{-1}e)$ ?
    – StubbornAtom
    1 hour ago














  • 1




    Isn't $det A=det B (1+e^TB^{-1}e)$ ?
    – StubbornAtom
    1 hour ago








1




1




Isn't $det A=det B (1+e^TB^{-1}e)$ ?
– StubbornAtom
1 hour ago




Isn't $det A=det B (1+e^TB^{-1}e)$ ?
– StubbornAtom
1 hour ago











2














Let $M_n$ be your matrix.



Let $eta_n$ be the $ntimes n$ matrix with entry $1$ at the superdiagonal and $0$ 4 elsewhere. If you




  1. Subtract row $k+1$ from row $k$ for $k = 1,2,ldots,n-1$.

    This is equivalent to multiply $M_n$ by $I_n - eta_n$ from the left


  2. Subtract column $k-1$ from column $k$ for $k = n,n-1,ldots,2$ (notice the order of $k$).

    This is equivalent to multiply $(I_n-eta_n)M_n$ by $I_n - eta_n$ from the right.



After you do this, your matrix simplfies to
$$(I_n - eta_n) M_n (I_n - eta_n) =
begin{bmatrix}
n-1&-n&0&cdots&0&0&0\
0&n-1&-n&cdots&0&0&0\
0&0&n-1&ddots&0&0&0\
vdots&vdots&vdots&ddots&ddots&vdots&vdots\
0&0&0&cdots&n-1&-n&0\
0&0&0&cdots&0&n-1&-lambda\
1&0&0&cdots&0&0&lambda-1
end{bmatrix}$$



From this, you can deduce



$$det[M_n] = det[(I_n - eta_n)M_n(I_n - eta_n)]
= (n-1)^{n-1}(lambda-1) + n^{n-2}lambda$$






share|cite|improve this answer























  • This is obviously the correct way since I checked the final answer. But the element "1" in the bottom left is still there and therefore not all elements (except diagonal and superdiagonal) are zeroes, and correct me if I am wrong, wouldn't that mean this is not a triangular matrix and we won't be able to find the det. by just multiplying the diagonal elements? And also I don't quite understand how we got (n^n-2 * λ)
    – Paul Vinur
    6 mins ago












  • @PaulVinur This is not a triangular matrix. Since the entries $a_{ij}$ vanishes unless $j = i text{ or } i+1 pmod n$. When you expand the determinant out completely, among all the $n!$ possible terms in the determinant, only two terms survive. i.e those of the form $prod_{i} a_ii$ and $prod_{} a_{i,i+1}$ ($i+1$ upto modulo $n$) survive. That's why the final expression is a sum of two terms.
    – achille hui
    1 min ago


















2














Let $M_n$ be your matrix.



Let $eta_n$ be the $ntimes n$ matrix with entry $1$ at the superdiagonal and $0$ 4 elsewhere. If you




  1. Subtract row $k+1$ from row $k$ for $k = 1,2,ldots,n-1$.

    This is equivalent to multiply $M_n$ by $I_n - eta_n$ from the left


  2. Subtract column $k-1$ from column $k$ for $k = n,n-1,ldots,2$ (notice the order of $k$).

    This is equivalent to multiply $(I_n-eta_n)M_n$ by $I_n - eta_n$ from the right.



After you do this, your matrix simplfies to
$$(I_n - eta_n) M_n (I_n - eta_n) =
begin{bmatrix}
n-1&-n&0&cdots&0&0&0\
0&n-1&-n&cdots&0&0&0\
0&0&n-1&ddots&0&0&0\
vdots&vdots&vdots&ddots&ddots&vdots&vdots\
0&0&0&cdots&n-1&-n&0\
0&0&0&cdots&0&n-1&-lambda\
1&0&0&cdots&0&0&lambda-1
end{bmatrix}$$



From this, you can deduce



$$det[M_n] = det[(I_n - eta_n)M_n(I_n - eta_n)]
= (n-1)^{n-1}(lambda-1) + n^{n-2}lambda$$






share|cite|improve this answer























  • This is obviously the correct way since I checked the final answer. But the element "1" in the bottom left is still there and therefore not all elements (except diagonal and superdiagonal) are zeroes, and correct me if I am wrong, wouldn't that mean this is not a triangular matrix and we won't be able to find the det. by just multiplying the diagonal elements? And also I don't quite understand how we got (n^n-2 * λ)
    – Paul Vinur
    6 mins ago












  • @PaulVinur This is not a triangular matrix. Since the entries $a_{ij}$ vanishes unless $j = i text{ or } i+1 pmod n$. When you expand the determinant out completely, among all the $n!$ possible terms in the determinant, only two terms survive. i.e those of the form $prod_{i} a_ii$ and $prod_{} a_{i,i+1}$ ($i+1$ upto modulo $n$) survive. That's why the final expression is a sum of two terms.
    – achille hui
    1 min ago
















2












2








2






Let $M_n$ be your matrix.



Let $eta_n$ be the $ntimes n$ matrix with entry $1$ at the superdiagonal and $0$ 4 elsewhere. If you




  1. Subtract row $k+1$ from row $k$ for $k = 1,2,ldots,n-1$.

    This is equivalent to multiply $M_n$ by $I_n - eta_n$ from the left


  2. Subtract column $k-1$ from column $k$ for $k = n,n-1,ldots,2$ (notice the order of $k$).

    This is equivalent to multiply $(I_n-eta_n)M_n$ by $I_n - eta_n$ from the right.



After you do this, your matrix simplfies to
$$(I_n - eta_n) M_n (I_n - eta_n) =
begin{bmatrix}
n-1&-n&0&cdots&0&0&0\
0&n-1&-n&cdots&0&0&0\
0&0&n-1&ddots&0&0&0\
vdots&vdots&vdots&ddots&ddots&vdots&vdots\
0&0&0&cdots&n-1&-n&0\
0&0&0&cdots&0&n-1&-lambda\
1&0&0&cdots&0&0&lambda-1
end{bmatrix}$$



From this, you can deduce



$$det[M_n] = det[(I_n - eta_n)M_n(I_n - eta_n)]
= (n-1)^{n-1}(lambda-1) + n^{n-2}lambda$$






share|cite|improve this answer














Let $M_n$ be your matrix.



Let $eta_n$ be the $ntimes n$ matrix with entry $1$ at the superdiagonal and $0$ 4 elsewhere. If you




  1. Subtract row $k+1$ from row $k$ for $k = 1,2,ldots,n-1$.

    This is equivalent to multiply $M_n$ by $I_n - eta_n$ from the left


  2. Subtract column $k-1$ from column $k$ for $k = n,n-1,ldots,2$ (notice the order of $k$).

    This is equivalent to multiply $(I_n-eta_n)M_n$ by $I_n - eta_n$ from the right.



After you do this, your matrix simplfies to
$$(I_n - eta_n) M_n (I_n - eta_n) =
begin{bmatrix}
n-1&-n&0&cdots&0&0&0\
0&n-1&-n&cdots&0&0&0\
0&0&n-1&ddots&0&0&0\
vdots&vdots&vdots&ddots&ddots&vdots&vdots\
0&0&0&cdots&n-1&-n&0\
0&0&0&cdots&0&n-1&-lambda\
1&0&0&cdots&0&0&lambda-1
end{bmatrix}$$



From this, you can deduce



$$det[M_n] = det[(I_n - eta_n)M_n(I_n - eta_n)]
= (n-1)^{n-1}(lambda-1) + n^{n-2}lambda$$







share|cite|improve this answer














share|cite|improve this answer



share|cite|improve this answer








edited 19 mins ago

























answered 53 mins ago









achille hui

95.3k5129256




95.3k5129256












  • This is obviously the correct way since I checked the final answer. But the element "1" in the bottom left is still there and therefore not all elements (except diagonal and superdiagonal) are zeroes, and correct me if I am wrong, wouldn't that mean this is not a triangular matrix and we won't be able to find the det. by just multiplying the diagonal elements? And also I don't quite understand how we got (n^n-2 * λ)
    – Paul Vinur
    6 mins ago












  • @PaulVinur This is not a triangular matrix. Since the entries $a_{ij}$ vanishes unless $j = i text{ or } i+1 pmod n$. When you expand the determinant out completely, among all the $n!$ possible terms in the determinant, only two terms survive. i.e those of the form $prod_{i} a_ii$ and $prod_{} a_{i,i+1}$ ($i+1$ upto modulo $n$) survive. That's why the final expression is a sum of two terms.
    – achille hui
    1 min ago




















  • This is obviously the correct way since I checked the final answer. But the element "1" in the bottom left is still there and therefore not all elements (except diagonal and superdiagonal) are zeroes, and correct me if I am wrong, wouldn't that mean this is not a triangular matrix and we won't be able to find the det. by just multiplying the diagonal elements? And also I don't quite understand how we got (n^n-2 * λ)
    – Paul Vinur
    6 mins ago












  • @PaulVinur This is not a triangular matrix. Since the entries $a_{ij}$ vanishes unless $j = i text{ or } i+1 pmod n$. When you expand the determinant out completely, among all the $n!$ possible terms in the determinant, only two terms survive. i.e those of the form $prod_{i} a_ii$ and $prod_{} a_{i,i+1}$ ($i+1$ upto modulo $n$) survive. That's why the final expression is a sum of two terms.
    – achille hui
    1 min ago


















This is obviously the correct way since I checked the final answer. But the element "1" in the bottom left is still there and therefore not all elements (except diagonal and superdiagonal) are zeroes, and correct me if I am wrong, wouldn't that mean this is not a triangular matrix and we won't be able to find the det. by just multiplying the diagonal elements? And also I don't quite understand how we got (n^n-2 * λ)
– Paul Vinur
6 mins ago






This is obviously the correct way since I checked the final answer. But the element "1" in the bottom left is still there and therefore not all elements (except diagonal and superdiagonal) are zeroes, and correct me if I am wrong, wouldn't that mean this is not a triangular matrix and we won't be able to find the det. by just multiplying the diagonal elements? And also I don't quite understand how we got (n^n-2 * λ)
– Paul Vinur
6 mins ago














@PaulVinur This is not a triangular matrix. Since the entries $a_{ij}$ vanishes unless $j = i text{ or } i+1 pmod n$. When you expand the determinant out completely, among all the $n!$ possible terms in the determinant, only two terms survive. i.e those of the form $prod_{i} a_ii$ and $prod_{} a_{i,i+1}$ ($i+1$ upto modulo $n$) survive. That's why the final expression is a sum of two terms.
– achille hui
1 min ago






@PaulVinur This is not a triangular matrix. Since the entries $a_{ij}$ vanishes unless $j = i text{ or } i+1 pmod n$. When you expand the determinant out completely, among all the $n!$ possible terms in the determinant, only two terms survive. i.e those of the form $prod_{i} a_ii$ and $prod_{} a_{i,i+1}$ ($i+1$ upto modulo $n$) survive. That's why the final expression is a sum of two terms.
– achille hui
1 min ago












Paul Vinur is a new contributor. Be nice, and check out our Code of Conduct.










draft saved

draft discarded


















Paul Vinur is a new contributor. Be nice, and check out our Code of Conduct.













Paul Vinur is a new contributor. Be nice, and check out our Code of Conduct.












Paul Vinur is a new contributor. Be nice, and check out our Code of Conduct.
















Thanks for contributing an answer to Mathematics Stack Exchange!


  • Please be sure to answer the question. Provide details and share your research!

But avoid



  • Asking for help, clarification, or responding to other answers.

  • Making statements based on opinion; back them up with references or personal experience.


Use MathJax to format equations. MathJax reference.


To learn more, see our tips on writing great answers.





Some of your past answers have not been well-received, and you're in danger of being blocked from answering.


Please pay close attention to the following guidance:


  • Please be sure to answer the question. Provide details and share your research!

But avoid



  • Asking for help, clarification, or responding to other answers.

  • Making statements based on opinion; back them up with references or personal experience.


To learn more, see our tips on writing great answers.




draft saved


draft discarded














StackExchange.ready(
function () {
StackExchange.openid.initPostLogin('.new-post-login', 'https%3a%2f%2fmath.stackexchange.com%2fquestions%2f3053935%2fdeterminant-of-an-n-x-n-matrix%23new-answer', 'question_page');
}
);

Post as a guest















Required, but never shown





















































Required, but never shown














Required, but never shown












Required, but never shown







Required, but never shown

































Required, but never shown














Required, but never shown












Required, but never shown







Required, but never shown







Popular posts from this blog

Михайлов, Христо

Гороховецкий артиллерийский полигон

Центральная группа войск